La ricerca ha trovato 297 risultati

da kalu
18 dic 2011, 13:32
Forum: Teoria dei Numeri
Argomento: Niente soluzioni
Risposte: 7
Visite : 2160

Re: Niente soluzioni

x^n , essendo congruo a -1 in modulo 3, non può essere un quadrato, quindi n è dispari. Sia p un divisore primo di n . \displaystyle \left(x^{\frac{n}{p}}+1\right)\left(\frac{x^n+1}{x^{\frac{n}{p}}+1}\right)=3^k . Chiaramente \displaystyle \left(x^{\frac{n}{p}}+1\right) e \displaystyle \left(\frac{...
da kalu
14 dic 2011, 14:28
Forum: Teoria dei Numeri
Argomento: Media quadratica intera
Risposte: 10
Visite : 2199

Media quadratica intera

Trovare tutte le terne di interi ($ a $, $b$, $c$) tali che $ 2c^2=a^2+b^2 $.
E' di un febbraio di qualche anno fa, qualcuno lo ricorderà...
da kalu
13 dic 2011, 16:10
Forum: Algebra
Argomento: Sistemi e soluzioni
Risposte: 7
Visite : 1948

Re: Sistemi e soluzioni

Per prima cosa poniamo x_1=a . Risolviamo innanzitutto il sistema costituito dalle prime n-1 equazioni, riservandoci solo alla fine di mettere il risultato a sistema con l'ultima equazione. Risolvendo le singole equazioni, vediamo che x_{i+1}=-x_i v x_{i+1}=x_i+1 \forall 1 \leq i < n . Consideriamo ...
da kalu
11 dic 2011, 13:33
Forum: Algebra
Argomento: Serie non troppo piccola
Risposte: 8
Visite : 2649

Re: Serie non troppo piccola

Mist ha scritto:la mia tesi si riduce a dimostrare che $\displaystyle x_{n+1} = x_{n}^2-x_{n}+1 > \frac{1}{n+1}$
Volevi dire $\displaystyle x_{n+1} = x_{n}^2-x_{n}+1 >1- \frac{1}{n+1}$ ;)
da kalu
10 dic 2011, 16:16
Forum: Teoria dei Numeri
Argomento: Numeri primi e quadrati perfetti
Risposte: 1
Visite : 1074

Re: Numeri primi e quadrati perfetti

Hai appena formulato un'interessante congettura: affermi che ogni quadrato possa essere espresso come media (aritmetica) di due numeri primi, ossia che per ogni intero n esistano due numeri primi p e q tali che n^2= \displaystyle \frac{p+q}{2} . (Molto) tempo fa un matematico di nome Goldbach formul...
da kalu
09 dic 2011, 14:28
Forum: Teoria dei Numeri
Argomento: 114. numeri perfetti
Risposte: 15
Visite : 4218

Re: 114. numeri perfetti

n^{-1}=1-\displaystyle \sum_{1 \leq i \leq k}{{d_i}^{-1}} \geq {t_k}^{-1} , dove t_k è il k -esimo termine della successione così definita: t_1=2 , t_{m+1}=t_m(t_m+1) . * *Bisogna dimostrare questa disuguaglianza, corollario dell'altro problema di Jordan (viewtopic.php?f=13&t=16466). Mi rendo c...
da kalu
08 dic 2011, 21:03
Forum: Teoria dei Numeri
Argomento: 114. numeri perfetti
Risposte: 15
Visite : 4218

Re: 114. numeri perfetti

Sia k=\sigma_0(n)-2 . Siano inoltre d_1 , d_2 , ..., d_{k} tutti i divisori positivi di n esclusi 1 e n . \displaystyle \sum_{1 \leq i \leq k}{d_i}=n-1 , quindi, dividendo ambo i termini per n , \displaystyle \sum_{1 \leq i \leq k}{{d_i}^{-1}}=1-n^{-1} . n^{-1}=1-\displaystyle \sum_{1 \leq i \leq k}...
da kalu
08 dic 2011, 14:09
Forum: Teoria dei Numeri
Argomento: gcd(ab+1,bc+1,ca+1)
Risposte: 10
Visite : 2337

Re: gcd(ab+1,bc+1,ca+1)

Più che d ∤ a serve ( a, d )=1 :shock: Giustissimo. Ma_go, è questo l'errore di cui parli? Se sì, riformulo ancora. d \mid (b-c)a . Se un divisore primo di d dividesse a , non potrebbe dividere ab+1 : assurdo dal momento che ab+1 è multiplo di d . Quindi b \equiv c (mod d ) I'm sorry :roll: (soprat...
da kalu
07 dic 2011, 19:55
Forum: Teoria dei Numeri
Argomento: Sommatorie notissime
Risposte: 8
Visite : 2561

Re: Sommatorie notissime

Io risolsi la sommatoria dei quadrati immaginando di contare i mattoni di una piramide costituita da basamenti quadrati sovrapposti uno sull'altro: a partire da quello più in basso di n per n mattoni, poi via via (n-1) per (n-1) , (n-2) per (n-2) ... fino al mattone finale posto all'apice. Feci un m...
da kalu
07 dic 2011, 19:37
Forum: Teoria dei Numeri
Argomento: gcd(ab+1,bc+1,ca+1)
Risposte: 10
Visite : 2337

Re: gcd(ab+1,bc+1,ca+1)

ma_go ha scritto:kalu ha scritto:
Ma se d∣ab+1 e d>1, d∤a: quindi b≡c (mod d).

questa riga mi piace molto poco.
E allora riformulo. $ ab \equiv -1 \not\equiv 0 $ (mod $ d $), da cui $ a \not\equiv 0 $ (mod $ d $). Va bene? :)
da kalu
06 dic 2011, 16:03
Forum: Teoria dei Numeri
Argomento: gcd(ab+1,bc+1,ca+1)
Risposte: 10
Visite : 2337

Re: gcd(ab+1,bc+1,ca+1)

Sia d=(ab+1, bc+1, ca+1) . Se d=1 la tesi è banale, dato che a+b+c \geq3 . Supponiamo ora d>1 . Dato che d \mid ab+1 e d \mid ca+1 , d \mid (b-c)a . Ma se d \mid ab+1 e d>1 , d \nmid a : quindi b \equiv c (mod d ). Ripetendo lo stesso ragionamento possiamo dimostrare che a , b e c sono tutti tra lor...
da kalu
06 dic 2011, 15:13
Forum: Teoria dei Numeri
Argomento: Primi e potenze
Risposte: 27
Visite : 5038

Re: Primi e potenze

Dimostriamo almeno che $ 4 $ non divide $ n+1 $
da kalu
05 dic 2011, 17:08
Forum: Combinatoria
Argomento: Dividiamo i sacchetti
Risposte: 2
Visite : 1888

Re: Dividiamo i sacchetti

Sia S il numero totale di palline. Siano x e y le quantità di palline presenti in due generici sacchetti. Voglio dimostrare per induzione che x \equiv y (mod 2^n ) \forall n (da cui seguirebbe immediatamente che x=y ). Caso n=1 . Escludiamo il sacchetto contenente x palline: rimangono S-x palline (c...
da kalu
05 dic 2011, 15:49
Forum: Teoria dei Numeri
Argomento: Primi e potenze
Risposte: 27
Visite : 5038

Re: Primi e potenze

Hawk ha scritto:$ a_n=b_n=0 $ da cui deduco p=0
Puoi spiegarti meglio? come deduci p=0?
da kalu
19 nov 2011, 15:55
Forum: Algebra
Argomento: Disuguaglianza
Risposte: 2
Visite : 1298

Re: Disuguaglianza

Provo a risolvere il massimo.
Per Cauchy Schwarz
$ (x+y+z)^2 \leq (x^2+2y^2+3z^2)(1+\frac{1}{2}+\frac{1}{3})=\frac{11}{6} $
L'uguaglianza vale quando $ x=3\sqrt{\frac{2}{33}}, y=\frac{3}{2}\sqrt{\frac{2}{33}}, z=\sqrt{\frac{2}{33}} $